Real Qual PDF

You might also like

Download as pdf or txt
Download as pdf or txt
You are on page 1of 35

Real Analysis qual study guide

James C. Hateley

1. Measure Theory
Exercise 1.1. If A R and  > 0 show open sets O R such that m (O) m (A) + .
[
Proof: Let {In } be a countable cover for A, then A
In . Since m (O) m (A) + . This implies
n=1

that
(

m (O)  m (A) where m (A) =

inf
S

In

)
l(In )

n=1

If l(Ik ) = for some k then there is nothing to show, so suppose (an , bn ) = In then l(In ) < , n.
Let On = (an + 2n , bn ) then we have
X

l(On ) = bn an 2n 
X
X
l(On ) =
bn an
2n  =
[
m ( On )  m (A)

l(In )
X
bn an 

So let O =

On , then m (O)  m (A) O R st m (O) m (A) + 

Exercise 1.2. If A, B R, m (A) = 0, then m (A B) = m (B)


Proof: m (A B) m (A) + m (B), and m (B) m (A B), hence we have
m (B) m (A B) m (A) + m (B) = m (B)
m (A B)

= m (B)

Exercise 1.3. Prove E M iff  > 0, O R open, such that E O and m (O\E) < 
Proof: () O\E = E c O implies that m (O\E) = m (E c O), but we have
m (O) = m (E c O) + m (E O)

So suppose m (E) < m (E c O) = m (O) m


S (E O). Let In be a countable cover for E, so
n
In = (an , bn ). Let On = (an , bn + 2 ) and let O = On . Then
X
X
X
m (O) =
l(On ) =
2n  + bn an =  +
bn an , and m (E O) = m (E)

since E O. So we have
=

m (E O)

m (E O) = m (E)

l(In )

bn an

X
l(On )
l(In )
X
X
+
bn an
bn an = 

O R open, st E O and m (O\E) 


() Conversely, suppose  > 0, O R, such that E O and m (O\E) <  and that O M. Then
m (O) = m (E c O) + m (E O), but m (E c O) = m (O\E) < 
This implies that
m (O) = m (E O) + 

m (O) = m (E) + 
1

EM

Exercise 1.4. Prove E M iff  > 0 F R closed, such that F E and m (E\F ) < 
Proof: () E\F = F c E this implies that m (E\F ) = m (F c E), but we have
m (F ) = m (F c E) + m (E F )
So suppose m (E) < m (F c E) = m (F ) m (FS E). Let In be a countable cover for E,
where In = (an , bn ). Let Fn = [an , bn 2n ] and let F = Fn . Then we have
X
X
X
m (F ) =
l(Fn ) =
bn an 2n  =
bn an ,
and m (E F ) = m (F ), since F E. So

X
X
m (E F ) = m (F )
l(In ) =
bn an
X
X
X
X
m (E F )
l(In )
l(Fn ) =
bn an
bn an +  = 

F R Closed, st F E and m (E\F ) 


() Conversely, suppose  > 0, F R, such that F E and m (E\F ) <  and that F M. Then
m (E) = m (F c E) + m (E F ),
but m (F c E) = m (E\F ) < . This implies that
m (E) m (F E) + 

m (E) m (F ) + 

EM

Vitali Let E be a set of finite outer measure and a collection of intervals that cover E in the sence
of Vitali. Then, given  > 0 there is a finite disjoint collection {IN } of intervals in such that
!
N
[

In < 
E\
n=1

Exercise 1.5. Does there exists a Lebesgue measurable subset A of R such that for every interval (a, b)
we have (A (a, b)) = (b a)/2?
Proof: First suppose that there is such a mesurable set A such that 0 6= (A (a, b)) = (b a)/2.
Then there exsits an open set O such that A O and (O\A) < , so let  = /2. Now O is open, so
there are disjoint intervals (xk , yk ) such that O is a countable union of these intervals. So

[
[
[(xk , yk ) (a, b)] = (ckl , dkl ).
O (a, b) =
k=1

Hence (O (a, b)) =

l dkl ckl , and we have

A O (a, b) = A (a, b) =

[A (ckl , dkl )]

Now
= (A (a, b)) =

1X
(dkl ckl )
2
l

but
X
(dkl ckl )

(O (a, b))

((O\A) (a, b)) + (A (a, b))


1X
(O\A) +
(dkl ckl )
2
l
1X
< +
(dkl ckl )
2
=

But this implies that


/2 = 

1X
(dkl ckl )
2
l

So (A) = 0. which implies that (Ac ) = . Now if there were to exsits such a set A we have (Ac ) = 0,
and so
1
b a = ((a, b)) = (A (a, b)) + (Ac (a, b)) = (Ac (a, b)) = (b a)
2
So there cannot exist such a set  .

Exercise 1.6. Assume that E [0, 1] is measurable and for any (a, b) [0, 1] we have
1
(E [a, b]) (b a)
2
Show that (E) = 1.
Proof: By the previous problem, using the same proof, we know that (E c ) = 0. So the result is
shown.
Exercise 1.7. Let E1 , . . . , En be measurable subsets of [0, 1]. Suppose almost every x [0, 1] belongs
to at least k of these subsets. Prove that atleast one of the E1 , . . . , En has measure of at least k/n.
Proof: Suppose not, then for each i we have (Ei ) < k/n. Define a function f (x) as follows.
n
X
Ei
f (x) =
i=1

where Ei denotes the characteristic function of Ei . Now since all most all x [0, 1] are in at least k
of the Ei we have f (x) k almost everywhere in [0, 1]. Now
Z
Z
n
n Z
X
X
Ei
Ei dx =
k=
k dx
f (x) dx =
[0,1]

[0,1]

i=1

[0,1]

i=1

But this implies that


n
X

Ei <

i=1

n
X
k
=k
n
i=1
k
n .

Which is a contradiction, hence at least one Ei has (Ei )

Exercise 1.8. Consider a measure space (X , A, ) and a sequences of measurable sets En , n N, such
that

X
(En ) <
n=1

Show that almost every x X is an element of at most finitely many En0 s.


Proof: It suffices to show that (x : x Enk ) = 0. So consider the following
!
m
\
lim x : x
Enk
m

k=1

If we have shown the above limit is zero, then were done. To see this look at the following sum,
!

X
\
X
x:x
Enk <
(En ) <
N =1

n=1

k=1

and hence
lim x : x

m
\

!
E nk

=0

k=1

Therefore almost every x X is an element of at most finitely many En0 s

.

Exercise 1.9. Consider a measure space (X , A, ) with (X ) < , and a sequences fn : X R of


measurable functions such that lim fn (x) = f (x) for all x X . Show that for every  > 0 there exists
n

a set E of measure (E)  such that fn converges uniformly to f outside the set E.
Proof: This is Ergoroffs theorem. See below.
Theorem (Egoroff s) If fn is a sequence of measurable functions that converge to a real-valued
function f a.e. on a measurable set E of finite measure, then given > 0, there is a subset A of E with
(A) < such that fn converges to f uniformly on E\A
Proof: Let > 0, then for each n, there exists a set An E with An < 2n , and there is an
Nn such that for all x
/ An and k An we have |fk (x) f (x)| < 1/n. Let A = An , then by
construction A E and A < . Choose n0 such that 1/n0 < . Now if x
/ A and k Nn0 then
|fk (x) f (x)| < 1/n0 < . Therefore fn converges uniformly on E\A.

Exercise 1.10. Let g be an absolutely continuous monotone function on [0, 1]. Prove that if E [0, 1]
is a set of Lebesgue measure zero, then the set g(E) = {g(x) : x E} R is also a set of Lebesgue
measure zero.
Proof: Let E [0, 1] with zero measure, then for any epsilon  > 0, there exists an open cover O for
E, such that (O\E) < . Now O being open in [0, 1] implies that O = (an , bn ), where (an , bn ) are
disjoint. Now by absolutely continuity of g(x) we have

X
X
> 0 s.t.
(In ) <

|g(In [0, 1])| <


n=1

n=1

Now g(E) |g(In [0, 1])| which implies that (g(E)) < , so given an there exists a > 0 such
that the above hold, then let = . Since is arbitrary we have (g(E)) = 0 
Remark: The above problem (1.10) is commonly refered to as Lusins N condition.

Exercise 1.11. Suppose f is Lipschitz continuous in [0, 1]. Show that


(a) (f (E)) = 0 if (E) = 0.
(b) If E is measurable, then f (E) is also measurable.
Proof: For part (a) if f is Lipschitz continuous then it is absolutely continuous, and so if (E) = 0,
then (f (E)) = 0 (see above proof).
For part (b) Let E be a measurable set and let  > 0. Now there exists an open set O such that
(O\E) < , where O is a disjoint union of intervals In = (an , bn ). Now since f is absolutely continiuous,
it can be approximated by simple functions, namely In . Choose these functions such that



X


cn In < 
f


n=1

Now (In ) = bn an > 0, so it is measurable. Let R, then the f (E) is measurable if {x : f (x) }
is a measurable set for any R. but we have now
{x : f (x) } {x : In +  }
We know simple functions are measurable, and our choice of simple functions approximates f (x),
therefore f is measurable  .
Theorem (Lusins) Let f be a measurable real-valued function on an interval [a, b]. Then given
> 0, there is a continuous function on [a, b] such that {x : f (x) 6= (x)} <

Proof: Let f (x) be measurable on [a, b] and let > 0. For each n, there is a continuous function hn
on [a, b] such that
{x : |hn (x) f (x)| 2n2 } < 2n2
Denote these sets as En . Then by construction we have
|hn (x) f (x)| < 2n2 , for x [a, b]\En
Let E = En , then E < /4 and {hn } is a sequence of continuous, thus measurable, functions that
converges to f on [a, b]\E. By Egoroffs theorem, there is a subset A [a, b]\E such that A < /4 and
hn converges uniformly to f on [a, b]\(EA). Thus f is continuous on [a, b]\(EA) with (EA) < /2.
Now there is an open set O such that (E A) O and (O\(E A)) < /2. Then we have f is continuous on [a, b]\O, which is closed. Hence there exists a that is continuous on (, ) such that
f = on [a, b]\O, where {x : f (x) 6= (x)} (O) <

Exercise 1.12. Prove the following statement. Supoose that F is a sub--algebra of the Borel -algebra
on the real line. If f (x) and g(x) are F -measurable and if
Z
Z
f dx =
g dx, A F
A

Then f (x) = g(x) almost everywhere.


Proof: Let denote the Lebesgue measure on the Borel sets. Now since both f and g are F -measurable,
for any n 1, the sets
An = {x : f (x) g(x) 1/n},

Bn = {x : g(x) f (x) 1/n}

are both measurable and contained in F . Now we also have

\
\
Bn
An , B = {x : g(x) f (x) > 0} =
A = {x : f (x) g(x) > 0} =
n=1

n=1

contained in F since F is a -algebra. Now using the convenetion that = 0, we have


Z
f g dx = 0
A
Z
If (A) > 0 then as f g > 0 implies by that
f g dx > 0, which is a contradiction. Hence we have
A

(A) = 0. By the same argument also have


Z
g f dx = 0

(B) = 0.

Now A B = and A B is the set of points where f (x) 6= g(x), hence f = g almost everywhere

.

Exercise 1.13. Let E R. Let E 2 = {e2 : e E}


(a) Show that if (E) = 0, then (E 2 ) = 0
(b) Suppose (E) < , it it true that (E 2 ) <
Proof: For part (a) consider the intervales In = [n, n+1] for in Z. Now consider the function f (x) = x2 .
If pn = (ak , bk ) is an open subset of In such that for < 0
(pn ) <

n
X
k=1

|f (bk ) f (ak )| =

N
X

|b2k a2k | (2|n| + 1)

k=1

Hence f (x) is absolutely continuous on In . Now a function is absolutely continuous on an interval I if


and only if the following are satisfies:
f is continuous on I

f is of bounded variation on I
f satisfies Lusins (N ) condition, or for every subset E of I such that (E) = 0, (f (E)) = 0.
Remark: The above condition for absolute continuity is the Banach-Zarecki Theorem.
Now define En = E In , then En In and hence by Lusins (N ) condition (f (En )) = 0. Now the set
f (En ) is given by
f (En ) = {e2 : e E In }
Now
E2 =

{e2 : e E In } =

nZ

f (En )

nZ

and so
(E 2 )

(En ) =

nZ

(En ) = 0

nZ

For part (b), the statement is not always true. For each n N, let En = [n, n + n3/2 ), then for each
(En ) = n3/2 . Now if E = En , then
(E) =

(En ) =

n=1

X
1
3/2
n
n=1

Now En2 = [n2 , n2 + 2n1/2 + n3 ), and so (En2 ) = 2n1/2 + n3 n1/2 . Also E 2 = En2 , and the
sets En2 are mutually disjoint. Hence
2

(E ) =

(En2 )

n=1

X
1

=
1/2
n
n=1

Exercise 1.14. Suppose a measure is defined on a -algebra M of subset of X , and is the


corresponding outer measure. Suppose A, B X . Then A B if (AB) = 0. Prove that is an
equivalence relation.
Proof: For symmetry we have, by definition, AB = (A B)\(A B) = (B A)\(B A) = BA,
and so if (AB) = 0, then (BA) = 0. Hence A B if and only if B A.
For reflexivity, we have (AA) = A\A = , hence A A.
For transitivity, let A, B, C X . First notice, by element chasing, AC (AB) (BC), and so
we have
0 (AB) = ((AB) (BC)) (AB) + (BC)
Now if A B and B C, then (AB) = (BC) = 0, and so (AB) = 0, hence A C.
Therefore is an equivalence relation on X  .

Exercise 1.15. Let (X , M, ) be a measure space.


(a) Suppose (X ) < . If f and fn are measurable functions with fn f almost everywhere,

[
prove that there exists sets H, Ek M such that X = H
Ek , where (H) = 0 and fn f
k=1

uniformly on each Ek
(b) Is the result of (a) still true if (X , M, ) is -finte?
Proof: For part (a), since (X ) < and fn f almost everywhere, by Egoroffs theorem, for any

k N, there is Hk M such that (Hk ) < 1/k and fn f uniformly on Ek = Hkc . Now define
H =
k=1 Hk , then H Hk , and so 0 (H) 1/k for all k, hence (H) = 0. Now
!c

[
[
\
c
Ek =
Hk =
Hk
= Hc
k=1

k=1

k=1

and so

X =H

!
Ek

k=1

where fk converges uniformly to f on any Ek

.

For part (b), the statement is true. Since X is -finite, we can write X as a disjoint union of finite sets,
i.e.

[
X =
Xn where (Xn ) < n Xi Xj = for i 6= j
n=1

Now for each Xn apply part (a). Then we have

Xn = Hn

Ek,n with (Hn ) = 0

k=1

Let H =
n=1 Hn , then (H) =

(Hn ) = 0. So we have

n=1

X =

Xn

Hn

n=1

n=1

!!
Ek,n

k=1

!
Hn

n=1

= H

Ek

n,k=1

Ek,n

n,k=1

Now H has measure zero and {Ek,n }


n,k=1 is a countable collection of open sets for which fn f
uniformly  .

Exercise 1.16. Suppose fn is a sequence of measurable functions on [0, 1]. For x [0, 1] define
h(x) = #{n : fn (x) = 0} (the number of indicies n for which fn (x) = 0. Assuming that h <
everywhere, prove that the function h is measurable.
Proof: First consider the measure space ([0, 1], [0, 1], ), where is the Lebesgue measure. Since fn
is measurable for all n we know that the set {x : fn (x) = } is measurable, for R. In particular,
the set {x : fn (x) = 0} is measurable. Now we have

{x : fn (x) = 0}

n=1

is measurable with respect to , since it is the countable union of measurable sets. Now consider the
measure space (N, (N), ) where is the counting measure. Now we know that
h(x) = {n : fn (x) = 0} <

So consider the following:


(
{x : h(x) = }



)

[


x : # fn (x) = 0 =

n

(
=

x:

n=1

x:

)
{n : fn (x) = 0} =
)
{n : fn (x) = 0} <

n=1

[0, 1]
Hence the function h(x) is measurable

.

2. Lebesgue Integration
Exercise 2.1. Consider the Lebesgue measure space (R, M, ). Let f be an extended real-valued M measurable function on R. For x R and r > 0 let Br (x) = {y R : |y x| < r}. With r > 0 fixed,
define a function g on R by setting
Z
g(x) =
f (y)(dy) for
xR
Br (x)

(a) Suppose f is locally -integrable on R. Show that g is a real-valued continuous function on R.


(b) Show that if f is -integrable on R then g is uniformly continuous on R.
Proof: If we show part (b), then part (a) follows by the same argument. Let x R. Now if f is
integrable on R2 so is |f |. Hence if  > 0, there is > 0 such that if (A) < , then we have
Z

|f | dy .
2
A
Now as B(x, r) and B(y, r) are open balls with area r2 with centers offset by |y x|, we have that
(B(x, r)\B(y, r)) = (B(y, r)\B(x, r)) 0 as y x
Hence given > 0, there is an > 0 such that if |y x| < , then
(B(x, r)\B(y, r)) = (B(y, r)\B(x, r)) <
So for |y x| < , we have
Z

|g(x) g(y)|

|f | d +
B(y,r)\B(x,r)

That is, g(x) is uniformly continuous on R

|f | d <
B(x,r)\B(y,r)



+ < .
2 2

.

Theorem (Jensens Inequality) If is a convex function on R and f an integrable function on


[0, 1].
Z

Z
(f (t)) dt
f (t) dt .
Proof: Let
Z
=

f (t) dt,

y = m(x ) + (a)

Then y is the equation of a supporting line at . Now we have


Z
(f (t)) m(f (t) ) + ()
(f (t)) dt () dt

Theorem (Bounded Convergence) Let fn be a sequence of measurable functions defined on a set


E of finite measure, and suppose that there is a real number M such that |fn | M for all N and all
x. If f (x) = lim fn (x) pointwise in E, then
Z
Z
f = lim
fn .
E

Proof: Let  > 0, thn there is an N and a measurable set A E with A <

n N and x E\A we have |fn (x) f (x)| < 2(E)
. Now,

Z
Z
Z






fn
f = fn f

E
E
ZE

|fn f |
ZE
Z
=
|fn f | +
|fn f |
E\A

<
Z

such that for all



+ =
2 2

Z
fn

Therefore we have


4M

.

Exercise 2.2. Suppose fn is a sequence of measurable functions such that fn converges to f almost
everywhere. If for each  > 0, there is a C such that
Z
|fn | dx < .
|fn |>C

Show that f is integrable on [0, 2]


Proof: First the interval [0, 2], is not important. The result can be shown for any finite interval. Fix
 > 0, now if f is to be integrable, then so is |f |. Let C be such in the hypothesis, by Fatous lemma
we have
Z 2
Z 2
|f | dx lim inf
|fn | dx
0
0
!
Z
Z
=

|fn | dx +

lim inf
[0,2]{|fn |>C}

|fn | dx
[0,2]{|fn |C}

 + C(0, 2)
Z

|f | dx is bounded and hence f is integrable

Therefore

.

Theorem (Fatous Lemma) If fn is a sequence of nonnegative measurable functions and fn (x)


f (x) almost everywhere on a set E, then
Z
Z
f lim inf
fn .
E

Proof: Since the integral over a set of measure zero is zero, (WLOG) we can assume that the converges
is everywhere. Let h be a bounded measurable fuction which is not greater that f and which vanishes
outside a set A E of finite measure. Define a function hn , by
hn (x) = min{h(x), fn (x)}.
Then hn is bounded by the bound for h and vanishes outside A. Now hn h pointwise in A, hence
we have by the bounded convergence theorem
Z
Z
Z
Z
h=
h = lim
hn lim inf
fn .
E

10

Taking supremum over h gives us the result

.

Theorem (Monotone Convergence) Let fn be an increasing sequence of nonnegative measurable


functions, and let f = lim f a.e. Then
Z
Z
f = lim fn .
Proof: By Fatous lemma we have
Z

Z
f lim inf

fn .
E

Now for each n, since f is monotone, we have fn f , and so


Z
Z
Z
Z
f
f

lim sup f
f

Z
f lim

fn

.

Remark: Let the positive part of f be denoted by f + (x) = max{f (x), 0}, and the negative part be denoted by f (x) = max{f (x), 0}. If f is measurable then so are f + and f . Futhermore f = f + f
and |f | = f + + f .

Exercise
R 2.3. Let f be a real-valued continuous function on [0, ) such that the improper Riemann
integral 0 f (x) dx converges. Is f Lebesgue integrable on [0, )?
Proof: f does not have to be Lebesgue integrable. Let n 0 and define a function fn as follows

1
4

n+1 x x [2n, 2n + 2 ]
4
1
fn (x) = n+1 x x [2n + 2 , 2n + 32 ]

4
3
n+1 x x [2n + 2 , 2n + 2]
Now fn is continuous on [0, ) and when considering Riemann integration, we have
Z 2n+1
Z 2n+2
Z
1
and
fn (x) dx =
fn (x) dx = 0
fn dx = 0
n+1
0
0
0
for each fixed n. Now define

X
fn (x)
f (x) =
n=0

Then since fn has disjoint support for any N N and 2N < y < 2N + 2, we have
Z y
Z y
f (x) dx =
f (x) dx,
0

2N

and so the Riemann integral of f (x) converges to 0 on [0, ). Now if a measurable function f is
Lebesgue integrablem then so is |f |. But,
Z

X
1
= .
|f | dx = 2
n+1
0
n=1
Therefore f is Riemann integrable but not Lebesgue integrable

.

Exercise 2.4. Consider the real valued function f (x, t), where x Rn and t I = (a, b). Suppose the
following hold.
(1) f (x, ) is integrable over I for all x E
(2) There exists an integrable function g(t) on I such that |f (x, t)| g(t), x E, t I.
(3) For some x0 E Zthen function f (, t) is continuous on I
Then the function F (x) =

f (x, t) dt is continuous at x0
I

Proof: Let xn be any sequence in E such that xn x0 . Define a sequence of functions as fn (t) =

11

f (xn , t). Then by hypothesis we have fn (t) g(t), for t I almost everywhere. Let f (t) = f (x0 , t),
now since f (x, t) is continuous at x0 , we have fn f . So by the Lebesgue Dominated Convergence
theorem we have
Z
lim
|fn (t) f (t)| dt = 0
n

Hence we have
Z
Z



|F (xn ) F (x0 )| = fn (t) f (t) dt |fn (t) f (t)| dt 0
I

Or F (x) is continuous at x0

Theorem (Lebesgue Dominated Convergence) Let g be integrable over E and let fn be a sequence
of measurable functions such that |fn | g on E and for almost all x E we have f (x) = lim fn (x).
Then
Z
Z
f = lim
fn .
E

Proof: Assuming the hypothesis, the function g fn is nonnegative, so by Fatous lemma we have
Z
Z
(g f ) lim inf (g fn )
E

Now since |f | g, f is integrable and we have


Z
Z
Z
Z
g
f
g lim sup
fn
E

Hence we have

Z
f lim sup

fn

Considering g + fn , we have the result


Z

Z
f lim inf

and so the result follows

fn
E

.

Exercise 2.5. Show that the Lebesgue Dominated Convergence theorem holds if almost everywhere
convergence is replaced by convergence in measure.
Proof: Suppose that fn f in measure, and there is an integrable function g such that fn g almost
everywhere. Now |fn f | is integrable for each n, and |fn f |[k,k] converges to |fn f |. By the
Lebesgue Dominated Convergence theorem we have
Z k
Z
|fn f |
|fn f |
k

Let  > 0, then there exsits an N0 such that


Z
|fn f | <
|x|>N0


3

also for each n, given  > 0, there exists > 0 such that for any set A with (A) < we have
Z

|fn f | <
3
A
Let A = {|fn f | }. Then there exists an N1 , such that for all n N1 , we have A = {|fn f |
} < . Let N = max{N0 , N1 }
Z
Z
Z
Z


|fn f | =
|fn f | +
|fn f | +
|fn f | < + + 2N < 
3 3
X
|x|>N
[N,N ]A
[N,N ]Ac
Z

Let =
, therefore we have
|fn f | 0, as n  .
6N
X

12

Exercise 2.6. Show that an extended real valued integrable function is finite almost everywhere.
Proof: Consider the measur space (X , M, ). Let E = {x C : |f | = }. Now since f is integrable,
it is measurable hence the set E is measurable. Now suppose (E) > 0, then as |f | > 0 on E we have
Z
Z
>
|f | d
|f | d =
X

This contradicts to the integrability of f , thus (E) = 0. Therefore f is finite almost everywhere

.

Exercise 2.7. If fn is a sequence of measurable functions such that


Z
X
|fn | <
n=1

Show that

fn converges almost everywhere to an integrable function f and that

n=1

Z
f=

Z
X

fn <

n=1

Proof: Define gN to be the partial sums of |fn |. Then gN is measurable since each fn , and hence |fn |
is measurable. Let g = lim gn , then g is measurable as it is the limit of measurable functions. Now
Z
Z X

Z
X
f=
|fn | =
|fn | <
n=1

n=1

So g is integrable, and hence g is finite almost everywhere. Define f (x) as follows


(P
R
if |g(x)| <
n=1 fn
f (x) =
0
otherwise
Then gN f as N almost everywhere. We also have
Z
Z


f
|f |



Z X




=
fn



n=1
Z X

|fn |
n=1

Z
=

g<

We also have that


N

N

X X
X


|gN | =
fn
|fn |
|fn | = g


n=1

n=1

n=1

almost everywhere. Now by the Lebesgue Dominated Convergence theorem, we have


Z

Z
f=

Z
lim gN = lim

gN = lim

N Z
X
n=1

fn =

Z
X
n=1

fn

13

Exercise 2.8. Let (X , M, ) be a measure space, and let fn be a sequences of nonnegative extended
real-valued M-measurable functions on X . Suppose lim fn = f exists almost everywhere on X and
fn f almost everywhere. For n N, show that
Z

Z
f d = lim

Proof: First if

f dx = , applying Fatous lemma we have


Z

lim inf fn d lim inf

X n

fn d lim

fn d .
X

And so lim

fn d =

fn d
X

f dx = .

R
Now if f dx < , since fn f almost everywhere, we have |fn | |f | almost everywhere, and we
have lim fn = f exists almost everywhere, we have by the Lebesgue Dominated Convergence theorem
Z

Z
| |fn | |f || d

|fn f | d = 0

Z
fn d =

lim

f d
X

Exercise 2.9. Let f be a nonnegative Lebesgue measurable function on [0, 1]. Suppose f is bounded
R1
above by 1 and 0 f dx = 1. Show that f = 1 almost everywhere on [0, 1]
Proof: let 1 >  > 0 and define the set E as
E = {x [0, 1] : 0 f 1 }
Now we have
Z
1 =

Z
f dx

f dx +
ZE

f dx
ZE
1  dx

f dx +
Ec

(E c ) + (E) (E)
=

1 (E)

Hence since this holds for any  (0, 1), we must have (E) = 0. Therefore f = 1 almost everywhere
on [0, 1]  .

Exercise 2.10. Let f be a real-valued Lebesgue measurable function on [0, ) such that:
(1) f is locally integrable
(2) lim f = c
x
Z
1 a
f dx = c.
Show that lim
a a 0

14

Proof: Let  > 0, then there is an M > 0 such that if x > M , then |f (x) c| < . Let a > M , now


Z a
Z


1
1 a


f c dx
f dx c =
a

a 0
0
Z
1 a

|f c| dx
a 0
Z
Z
1 M
1 a
=
|f c| dx +
|f c| dx
a 0
a M
Z
1 M
1
<
|f c| dx +  (a M )
a 0
a
Z
M
1 M
|f c| dx + (1
=
)
a 0
a
Now since M is fixed, and by the integrability of f , we have
Z a

1


f dx c < 
a
0

and since this is for any  > 0 and all a > M , we have
Z a

1


lim
f dx c = 0
a a
0

1
a a

lim

f dx = c

.

Exercise 2.11. Let f be a real-valued uniformly continuous function on [0, ). Show that if f is
Lebesgue integrable on [0, ), then lim f (x) = 0.
x

Proof: First if f is Lebesgue integrable, then so is |f |. Now decompose the integral as follows
Z
Z k+1
Z k+1
X
>
|f (x)| dx =
|f (x)| dx, denote ak =
|f (x)| dx.
0

k=1

Now ak > 0, and since the integral is convergent this implies that ak 0 as k , which inturn
implies that ak is Cauchy. So we have


Z
m
X



 > 0, N s.t.
ak < , n, m > N

|f (x)| dx < 


N +1
k=n

Since |f (x)| is positive and  is arbitrary this implies that f (x) 0 as N

.

Exercise 2.12. Let f L1 (R). With h > 0 fixed, define a function h on R by setting
Z x+h
1
f (t) (dt), for x R
h (x) =
2h xh
(a) Show that h is measurable on R.
(b) Show that h L1 (R) and kh k1 kf k1 .
For part (a) since f is integrable, then f is measurable. So the integral of a measurable function is
measurable, thus h (x) is measurable.
For part (b) First apply the change of variable y = x t, then we have
Z h
Z
Z x+h
Z h
f (t) (dt) =
f (x y) (dy) =
f (x y) (dy) =
xh

f (x y)[h,h] (y) (dy)

Where [h,h] (y) is the charactistic function on [h, h]. So we have


h (x) =

1
f [h,h]
2h

kh (x)k1 =

1
1
kf [h,h] k1
kf k1 k[h,h] k1 = kf k1
2h
2h

.

15

Exercise 2.13. Let f be a Lebesgue integrable function of the real line. Prove that
Z
lim
f (x) sin(nx) dx = 0.
n

Proof: If f is intergrable, then there exists a sequences of step function n such that
Z

 > 0 N s.t. |f n | <
2
Now we have

Z


f (x) sin(nx) dx


R

Z
|f (x) sin(nx)| dx
ZR

|(f (x) n (x)) sin(nx)| dx +


Z

+ |n (x) sin(nx)| dx
2
R
R

<

|n (x) sin(nx)| dx
R

Now
S n being a step function we have it as the sum of simple functions over disjoint interval In , where
n=1 In = R, i.e.
n =

ak,n Ik,n

k=1

and so we have
Z

Z
|n (x) sin(nx)| dx

|ak,n |

k,n


sin(nx) dx

Z
|ak,n |

|sin(nx)| dx 0 as n
Ik,n

k=1

Hence for some N large enough and all n > N we have



Z
Z




f (x) sin(nx) dx <  +
|n (x) sin(nx)| < + = 
2

2
2
R
R

3. Convergence
Exercise 3.1. Consider the Lebesgue measure space (R, M, ) on R. Let f be a -integrable extended
real-valued M-measurable function on R. Show that
Z
|f (x + h) f (x)| (dx) = 0.
lim
h0

Proof: First since f (x) is integrable, we have


Z
Z
f (x + h) (dx) =
f (x) (dx)
R

h R

Also since f is integrable, there exists a sequence of continuous function n , such that
Z

|f (x) n (x)| (dx) <
3
Now |n (x + h) n (x)| < 3 if |h| < . Let N be large enough, then
Z
Z
  
|f f (x+h)| (dx) |f n (x)|+|n (x+h)f (x+h)|+|n (x+h)n (x)|(dx) < + + = 
3 3 3
Therefore we have
Z
|f (x + h) f (x)| (dx) = 0

lim

h0

.

16

Exercise 3.2. Let (X , M, ) be a measure space. Let fn and f be an extended real-valued M- measurable fuctions on a set E X such that lim fn = f on E. Then for every R we have
n

{E : f > } lim inf {E : fn } and {E : f < } lim inf {E : fn }


n

Proof: I will only show the first inequality since the proofs are identical. Let A = {x E : f (x) },
A,n = {x E : fn (x) } and let A denote the characteristic function of A. First we need to show
An A in measure. Let  > 0, denote the set F,n by
F,n = {x E : |A,n A | }.
Now we want to show that the measure of this set is small. First notice that
c
F,n
{x A : |f fn | < }

Let x be in this subset, then this implies two thing. First if f (x) > > , and fn (x) > f (x) > .
So we must have
F,n {x A : |f fn | }
Now since fn converges to f almost everywhere in E, it converges in measure, and hence the measure
of the set (F,n ) < . This implies that A,n converges to A in measure. Now Fatous lemma
holds for a sequence of functions converging in measure, so we have
Z
Z
A d lim inf
A,n d {E : f > } lim inf {E : fn } 
E

Exercise 3.3. Let g(x) be a real-valued function of bounded variation on an interval [a, b]. Suppose
that f is a real-valued decreasing function on [a, b]. Show that g(f (x)) is also of bounded variation. If
f is just a bounded continuous function is g(f (x)) still of bounded variation.
Proof: Since g is of bounded variation we have, let P be all the possible partitions of [a, b]
Vab (g) = sup

n
X

|g(xi ) g(xi1 )|

{xi }P i=1

Now fix  > 0 and pick an {xi } such that


Vab (g) <

N
X

|g(xi ) g(xi1 )| + 

i=1

Now since f is decreasing on we have that f (xi+1 ) < f (xi ). Now call yi = fxi , {yi } {a, b} then is a
partition of [a, b], and so we have
N
X
i=1

|g(yi ) g(yi1 )| < sup

n
X

{xi }P i=1

|g(xi ) g(xi1 )| = Vab (g)

This can be done for any partition of [a, b].Therefore g(f (x)) is also of bounded variation.
For the second part, no. Consider the function
(
x sin
f (x) =
1

1
x

x 6= 0
x=0

and let g(x) = x. Now g(x) is a function of bounded variation on [1, 1] and f (x) is a bounded and
continuous on [1, 1], but g(f (x)) = f (x), which not a function of bounded variation on [1, 1].

Exercise 3.4. Let (X , M, ) be a measure space. Let fn and f be an extended real-valued M- measurable fuctions
Z on a set E X with (E) < . Show that fn converges to 0 in measure on E if and
|fn |
only if lim
d = 0
n E 1 + |fn |

17

Proof: () If fn converges to 0 in measure then we have


{x E : |fn | } < .
Call this set A . Now
Z
E

|fn |
d =
1 + |fn |

Z
A

|fn |
d +
1 + |fn |

Z
Ac

|fn |
d
1 + |fn |

1
1+x , x

0 is monotone, and uniformly continuous on any bounded interval. Now


Now the function
(A ) < , and so there is a such that So we have
Z
Z
|fn |
|fn |
d +
d < (A ) + (E) < (1 + (E))
A 1 + |fn |
Ac 1 + |fn |
Hence we have

Z
E

|fn |
d 0 as  0
1 + |fn |

() Now suppose that


|fn |
d 0 as  0
E 1 + |fn |
and suppose that there exists and 0 such that {x E : |fn | 0 } 0 . Then we have
Z
Z
Z
|fn |
20
|fn |
|fn |
d +
d
+
d
2
1 + 0
Ac 1 + |fn |
Ac 1 + |fn |
A0 1 + |fn |
Z

20
1 + 20

But this implies that


20

1 + 20
let  =

20
2(1 + 20 )

Z
A0

|fn |
d +
1 + |fn |

, then we have a contradiction

Z
Ac

|fn |
<
1 + |fn |

.

Exercise 3.5. Suppose (E) < and fn converges to f in measure on E and gn converges to measure
on E. Prove that fn gn converges to f g in measure on E.
Proof: Let hn = fn gn and let h = f g. Now h and hn are measurable since fn and gn are. For each
> 0 define
An () = {x : |hn (x) h(x)| }
and let an () = (An ()). Now because fn and gn converge in measure, for any subsequences fnk , gnk
there are subsequences fnkj and gnkj , such that both fnkj and gnkj converge almost everywhere to f
and g respectively. Hence we have hnkj = fnkj hnkj , which converges to h = f g almost everywhere
on E. Now since hnkj converges almost everywhere and (E) is finite we have that hnk converges in
measure. Now
lim |h hn | lim sup |h hnk | 0
n

k n

Hence lim an () = lim ank () = 0, or hn converges in measure


n

.

(Convergence in measure) A sequences fn of measurable functions is said to converge to f in


measure if, given  > 0, there is an N such that for all n N we have
{x : |f (x) fn (x)| } 
Remark: Let an be a sequence of real numbers. If there is an a R, such that for every subsequence
ank , there is a subsequences for which ankl a, then an a.

18

Exercise 3.6. If fn , f L2 and fn f almost everywhere, then kfn f k2 0 if and only if


kfn k2 kf k2 .
Proof: () Suppose kfn f k2 0, now
kfn f k22

Holders inequality

fn2 2f fn + f 2
Z
2
kf k2 2 |fn f | + kf k22

kf k22 2kfn k2 kf k2 + kf k22

|kf k2 kfn k2 |2

Therefore as kfn f k22 0 we have kfn k2 kf k2 .


() Now suppose kfn k2 kf k2 and fn f almost everywhere. Now for p 1, and for finite a,b, we
have
|a + b|p 2p (|a|p + |b|p )
For each n, let
gn = 4(|fn |2 + |f |2 ) |fn f |2 .
Now gn 0 almost everywhere. Since fn and f are finite almost everywhere, by Fatous lemma we
have
Z
Z
lim inf gn lim inf gn
2
Now since fn
Z f almost everywhere we have lim inf gn = 8|f | almost everywhere. So we have
8kf k22 lim inf gn . Now
Z
Z
Z
Z
lim inf gn = 4 lim inf |fn |2 + 4 lim inf |f |2 = lim sup |fn f |2

4 lim inf kfn k22 + 4kf k22 lim sup kfn f k22

8kfn k22 lim sup kfn f k22

so we have 0 lim sup kfn f k22 , hence 0 lim sup kfn f k22 0. Therefore we have
lim sup kfn f k2 = lim inf kfn f k2 = 0

kfn f k2 0

.

Remark: A sequences of functions fn converges in measure to f if and only if for every sequences fnk ,
there is a subsequence fnkj that converges almost everywhere to f .

Exercise 3.7. If fn 0 and fn (x) f (x), in measure then


Z
Z
f (x) dx lim inf fn (x) dx
Proof: Let fnk be any subsequence of fn . then there exists an fnkj such that fnkj converges to f
almost everywhere. By Fatous Lemma we have
Z
Z
Z
Z
f lim inf fnkj = lim fnk lim inf fn
Exercise 3.8. Suppose fn converges to two functions f and g in measure on D. Show that f = g
almost everywhere on D
Proof: Define the set E as E = {x D : |fn (x) f (x)| > 0}. Then if En = {x D : |fn (x) f (x)|
1
1
and |fn (x) g(x)| < 2m
,
1/m}, we have E = lim En . Now if for some n we have |fn (x) f (x)| < 2m
then we have
1
|f g| |fn f | + |fn g| <
m

19

And so

x : |fn (x) f (x)| <

1
2m



1
x : |fn (x) g(x)| <
2m

x : |f (x) g(x)| <

1
2m

which implies that


 




1
1
1
x : |fn (x) g(x)|

x : |f (x) g(x)|
x : |fn (x) f (x)|
2m
2m
2m


1
2
2
This implies that x : |f (x) g(x)|
0. Hence
<
. Now as n , we have m
2m
m
{x : |f (x) g(x)| > 0} = 0  .

Exercise 3.9. Let fn f in Lp (X , M, ), with 1 p < , and let gn be a seqences of measurable


functions such that |gn | M < for all n, and gn g almost everywhere. Prove that gn fn gf in
Lp (X , M, )
Proof: Since fn f in Lp , since Lp is complete we have f Lp . Also since |gn | M , for all n this
implies that |g| M . Now
Z
Z
kfn gn gn f kpp = (fn gn gn f )p M p |fn f |p
M p kfn f kpp
So we have kfn gn gn f kp M kfn f kp , and so
kfn gn gf kp M kfn f kp 0 as n
Therefore gn fn gf in Lp (X , M, )

.

Exercise 3.10. Suppose f is differentiable everywhere on (a, b). Prove that f 0 is a Borel measurable
function on (a, b)
Proof: f 0 is Borel measurable if {x : f 0 (x) } is a Borel set. So
 


1
f 0 (x) lim n f x +
f (x)
n
n


 

1
f (x) 0
lim f x +
n
n
n
 


1

1
for all but finitely many n
f x+
f (x)
m
n
n
m




1

1
x lim inf x : f x +
f (x)
m
n
n
m



[ \
1

1
x
x:f x+
f (x)
m
k
k
m
n1 kn



\ [ \
1

1
x
x:f x+
f (x)
k
k
m
m1 n1 kn

Now since f (x) is differentiable almost everywhere, it is continuous almost everywhere and so the f (x),
and f (x + 1\k) are measurable. Any linear combination of them is measurable, and so the set




1

1
x:f x+
f (x)
k
k
m
is measurable. Now the collection of all such sets form a -algebra, and hence the countable union and
intersection of these sets are measurable. Therefore f 0 (x) is measurable  .

20

Exercise 3.11. Let cn,i be an array of nonnegative exteneded real numbers for n, i N.
(a) Show that
lim inf

cn,i

iN

X
iN

lim inf cn,i

(b) If cn,i is an increasing sequences for each i N then


X
X
lim
cn,i =
lim cn,i
n

iN

iN

Proof: For part (a) first let denote the counting measure. Now if M = P(N), then (N, M, )
forms a measure space. Now let an be sequences with cn [0, ]. Then the function a(n) = an is
M-measurable, and so
Z
X
c d =
cn
nN

Then by Fatous lemma we have


Z
Z
lim inf cn lim inf
cn
N n

nN

X
iN

lim inf cn,i lim inf

cn,i

iN

For part (b) using the same measure space (N, M, ), we know that cn (i) cn (i+1), so by the Monotone
convergence theorem we have
Z
Z
X
X
lim cn = lim
cn

lim cn,i lim


cn,i 
N n

iN

iN

Theorem (Ascoli-Arzela) Let F be an equicontinuous family of functions from a separable space


X to a metric space Y . Let fn be a sequence in F such that for each x X the closure of the set
{fn (x) : 0 n < } is compact. Then there is a subsequence fnk that converges pointwise to a
continuous function f , and the convergence is uniform on each compact subset X.
Exercise 3.12. Let {qk } be all the rational numbers in [0, 1]. Show that

X
1
1
p
converges a.e. in [0, 1]
k 2 |x qk |

k=1

Proof: Fix 0 > 0, consider the two sets


1
1
E1 = p

0
|x qk |

1
1
and E2 = p
>
0
|x qk |

Now for each fixed x [0, 1]\Q we can enumerate the rationals however we want (Zorns Lemma).
Choose such an ordering so that
1
x E2
p
< k 10
|x qk |
That is the closer qk gets to x, the large the index. Now let be the counting measure, then we have
Z
Z

X
1
1
1
1
1
1
p
p
p
=
d
+
d
2
2
2
k
|x qk |
|x qk |
|x qk |
E1 k
E2 k
k=1
Z
Z
1 1
1
<
+
d
2 
1+0
k
k
0
E
E
Z 1
Z 2
1 1
1
<
+
d <
2 
1+0
k
k
0
N
N
This can be done for all x [0, 1]\Q. Therefore the series converges almost everywhere in [0, 1]

.

21

Exercise 3.13. Let (X , M, ) be a finite measure space. Let fn be an arbitrary sequence of real-valued
measurable functions on X . Show that for every  > 0 there exists E M with (E) <  and a sequence
of positive real numbers an such that an fn 0 for x X \E
Proof: First denote the set Em = {x : m 1 |fn | < m}, then the sets Em are disjoint and cover X .
Now define as such

X
1
=
k2
k=1

Since (X ) < , if  > 0, there is an Mn such that


X

(En ) = {x : |fn | Mn }
>
n2
mMn

Now choose these Mn such that Mn > Mn1 for all n. Define the sets Fn = {x : |fn | Mn }, then we
have (Fn ) < n 2 . Now if E = En , then
(E)

(En ) <

n=1

 X 1
=
n=1 n2

Let an = 1/Mn3 , then if x X \E, then we have


an |fn (x)| <
And so we have

1
1
Mn = 2
Mn3
Mn

X
X
X
X
1
1


|an fn (x)|
an fn (x)

<

2


Mn
n2
n=1
n=1
n=1
n=1

Therefore we must have an fn (x) 0 on X \E

.

Exercise 3.14. Prove that the gamma function


Z
(x) =

tx1 et

is well defined and continuous for x > 0


Proof: Let let f (t, x) = tx+1 et , and x > 0 and decompose the integral into two integrals (0, 1] and
(1, ). For the first we have
Z 1
Z 1
tx x
x1 t
t
e dt
tx1 dt = 0 <
x
0
0
Now f (t, x) is continuous on (1, ), and also t2 f (t, x) 0 as t , so there is an M such that M
bounds t2 f (t, x) on (1, ). Now
Z
Z 1
Z
1
tx1 et dt =
tx+1 et t2 dt = M
dt = M
2
t
1
0
0
And so (x) is well defined on (0, ).
To show continuity, let xn , be a cauchy sequence, and define fn (t) = f (t, xn ). Now by continuity of
f (t, x) on (0, ) (0, ), we have that for each x, fn f on t (0, ). now f (t, x) is bounded on
(1, ), call this bound M > 1. Define a function g(t) by
(
tx1
0<t1
g(t) = M t
t e
1<t
Now fn , f g on (0, ), so by the Lebesgue Dominated Convergence theorem we have
Z
|fn f | 0 as n
0

22

and so we have
Z

|(xn ) (x)| =

Z

fn (t) f (t, x) dt

fn (t) f (t, x)| dt 0 as n

This holds for any sequence such that xn x (0, ), therefore (x) is continuous on (0, )

.

4. Lp spaces
Exercise 4.1. Let 1 p < q < . Which of the following statements are true and which are false?
(a)
(b)
(c)
(d)

Lp (R) Lq (R)
Lq (R) Lp (R)
Lp ([2, 5]) Lq ([2, 5])
Lq ([2, 5]) Lp ([2, 5])

Proof: Only part (d) is true. This can easily be shown for any finite interval, let I = [a, b] Let
f Lq (I). Then |f |p Lq/p (I). Now by Holders inequality we have
Z
|f |p = k|f |p kq/p k1kr
I

where r is conjugate to pq . Now


kf kpp k|f |p kq/p k1kr =
Hence we have kf kp kf kq (I)

qp
qp

Z

(|f |p )q/p

p/q
(I)

qp
q

= kf kpq (I)

qp
q

, therefore f Lp (I)

.

For a counterexample to part (c) consider the function f (x) = (x 2)1/2 , and let p = 1 and q = 2,
then f Lp ([2, 5]), but f
/ Lq ([2, 5]).
For a counterexample to part (b) consider the function f (x) = (1 + x2 )1/2 , and let p = 1, q = 2, then
f Lq (R) but f
/ Lp (R).
For a counterexample to part (a) consider the counterexample to part (c) with the zero extension.
Theorem (Holder Inequality) If p and q are nonnegative extended real numbers such that
1 1
+ =1
p q
and if f Lp and g Lq then f g L1 and
Z
|f g| kf kp kgkq
Proof: Assume 1 < p < , and suppose that f , g 0. Let h = g q1 , then g = hp1 . Now
ptf (x)g(x) = ptf (x)hp1 (h(x) + tf (x))p h(x)p
so we have

Z
pt

Z
fg

and we also have

Z
pt

|h + tf |p

hp = kh + tf kpp khkpp

f g khkpp + ktf kpp khkpp

now differentiating bothsides with respect to t at t = 0 , we have


Z
p f g pkf kp khkp1
= pkf kp kgkq
p

.

23

Exercise 4.2. Let f L3/2 ([0, 5]). Prove that


t

lim

t0+ t1/3

f (s) ds = 0.
0

Proof: Applying Holders inequality we have




Z t
1



f
(s)
ds
t1/3

|f (s)| ds

t1/3

Z

2/3 Z
|f (s)| ds

1/3
ds

t1/3
0
0
1
1/3 kf (s)k3/2 t1/3
t
2/3
Z t
|f (s)|3/2 ds
0 as

t0+

Exercise 4.3. Suppose f C 1 [0, 1], f (0) = f (1), and f > f 0 everywhere.
(1) Prove that f > 0 everywhere.
(2) Prove that
Z 1
Z 1
f2
d

f d
0
0 f f
0
Proof: For (1), if f (x) = R+ then everything holds. So suppose that, there exists an c (a, b)
(0, 1) such that f 0 (c) = 0. (WLOG) suppose that this c is not a saddle point for f (x), also suppose that
f (c) < 0. Now if there is a > 0 such that f (c) > f (x), for all x B(c, ), then we have f 0 (x) > 0 for
x (c , c). This implies that f 0 (x) > f (x) for x (c , c). If there is a > 0 such that f (c) < f (x),
for all x B(c, ), then we have f 0 (x) > 0 for x (c, c + ), which implies that f 0 (x) > f (x) for
x (c, c + ). For both cases we have a contradiction. Therefore f (x) > 0 for all x (0, 1). Now if
f (0) = 0, f cannot be constant since 0 6 0. this implies that, for some > 0, f 0 (x) > 0 for x [0, ),
which is a contradiction. Therefore f (x) > 0 for all x [0, 1].

For (2) since f > f 0 we have that f f 0 is well defined on [0, 1]. So,
Z

2

2
p
f
0 d
f

f
f f0
0
Z 1
Z 1
f2

d
f f 0 d
0
f

f
0
0
Z 1
Z 1
f2
d
f d

0
0
0 f f
Z

d =

H
olders inequality

The last line holds since f f 0 > 0. This implies that:


Z

Z
f d

f2
d
f f0

Exercise 4.4. If f (x) Lp L for some p < . Show that


(a) f (x) Lq for q > p.
(b) limq kf kq = kf k .
Proof: For part (a) Let 0 < p < q < and let f Lp L . Then if =

q
p

and if =

q
qp ,

then we

24

have

= 1. Now applying Holders inequality we have


Z
kf kqq =
|f |q
Z
1
1
=
|f |q( + )
Z
=
|f |p |f |qp
Z
=
|f |p |f |qp

k|f |p k1 k|f |qp k

Now since |f | kf k almost everywhere and q p > 0 we have |f |qp k|f |qp k almost everywhere,
and so k|f |qp k < . Also since f is monotone increasing, we have k|f |qp k = kf kqp
. We also
have k|f |p k1 = kf kpp < . Therefore f Lq  .
For part (b), first suppose that kf k = 0. This implies that f = 0 almost everywhere and hence
kf kq = 0 for all q. Hence limq kf kq kf k trivially.
Now suppose that f Lp L and kf k =
6 0. From part (a) we have
1/q
1 p
kf kq kf kpp
(kf k ) q
Now let  > 0, then on a set E of nonzero measure, |f | > kf k . If (E) = , shoose a subset of E
with finite measure. Then we have
Z
q
kf kq =
|f |q d
ZE

(kf k )q d
E

= (E)|kf k |q .
Now this is for all q > p. Let qn be a sequence of numbers greater than p that converges to . Then
1

lim (E) qn |kf k |

lim inf kf kqn

lim sup kf kqn

lim sup kf kpp

 q1

1 qpn

(kf k )

and so
|kf k | lim inf kf kqn lim sup kf kqn kf k
n

Since this holds for all  > 0 we have limn kf kqn = kf k . Now since this is for any sequence qn , we
have limq kf kq = kf k .

Exercise 4.5. Suppose that f Lp ([0, 1]) for some p > 2. Prove that g(x) = f (x2 ) L1 ([0, 1])
Proof: f Lp ([0, 1]) implies that kf kp < . In particular this implies that kgkp = kf (x2 )kp < .
Now
Z 1
Z 1
|g(x)| dx =
|f (x2 )| dx
0

change of variables (y = x2 )

h
olders inequality

1
|f (y) | dy
2 y
0



1
1

kf kp

y p
2
p1

25



Now f Lp ([0, 1]) and since p > 2 we have 1y

p
p1

< , therefore g(x) L1 ([0, 1])

.

Exercise 4.6. Let f Lp (X ) Lq (X ) with 1 p < q < . Prove that f Lr (X ) for all p r q.
Proof: Let E1 = {x : 0 |f (x)| 1}, and E2 = {x : 1 > |f (x)|}, then E1 , E2 are a Hahn
decomposition for X . Now suppose f Lp Lq . Now
Z
Z
r
r
kf kr =
|f | +
|f |r
E1
E2
Z
Z
p

|f | +
|f |q
E1
E2
Z
Z

|f |p +
|f |q
X

= kf kpp + kf kqq

f Lr (X )

Exercise 4.7. Suppose f and g are real-valued -measurable functions on R, such that
(1) f is -integrable.
(2) g C0 (R).
For c > 0 define gc (t) = g(ct). Prove that:
Z
(a) lim
f gc d = 0,
c R
Z
Z
(b) lim
f gc d = g(0) f d.
c0

Proof: For part (a) define hn (x) = f (x)gn (x). Now since f L1 (R) we know that f (x) < a.e., and
since g C0 (R) we know that
gn (x) 0 as n .
For a fixed x such that f (x) < we have
hn (x) 0 as n
Hence hn 0a.e.. Also since g C0 (R) we have that there is some M such that |g(x)| < M . So we
have
Z
Z
Z


hn (x) d
|f (x)gn (x)| d M
|f (x)| d <


R

since f L1 (R). Hence by the Lebesgue Dominated Convergence theorem we have


Z
Z
Z
lim
f gn d = lim
hn d =
lim hn d = 0
n

R n

Proof: For part (b) we know that for all n > 0, f gn L1 (R). Define hn (x) = |f (x)g(xn1 )|, again
since g C0 (R) we have that there is some M such that |g(x)| < M . So
Z
Z
Z


hn (x) d
|f (x)gn (x)| d M
|f (x)| d <


R

Hence by the Lebesgue Dominated Convergence theorem we have


Z
Z
lim
f g1/n d = lim
hn d
n R
n R
Z
=
lim hn d
n
ZR
=
lim f g1/n d
R n
Z
= g(0) f d
R

26

Exercise 4.8. Let E be a measurable subset of the real line. Prove that L (E) is complete.
Proof: Let fn be a Cauchy sequence of measurable functions in L . Then there exists and k N such
that if m, n Nk , then
1
1
kfn fm k < , n, m > Nk
|fn fm | < a.e.
k
k
Now define the sets En,m,k by


1
En,m,k = x E : |fn (x) fm (x)|
k
then for each n, m > Nk , the set En,m,k is empty. Let F be defined by
[
F =
En,m,k
km,n,Nk

Now F is a countable union of empty sets, and therefore is empty. Now for any x E\F we have
1
|fn (x) fm (x)| <
k
and so fn (x) is a Cauchy sequence in R. Now
1
|fm (x)| |fm (x) fn (x)| + |fn (x)| < + |fn (x)|.
k
Taking m , we have
1
1
|f (x)| + |fn (x)| < + kfn (x)k a.e.
k
k
Hence for each n we have |f | k1 + kfn k almost everywhere so f L . Therefore L is complete
.
Theorem (Riesz-Fischer) The Lp (E) spaces are complete.
Proof: For 1 p < , let fn be a Cauchy sequence on Lp .
 > 0 N s.t. kfm fn kp <  n, m > N
Now let nk = N 2k , then the subsequence fnk , satisfies
kfnk+1 fnk kp <

1
2k

Define the function f by


f (x) = fn1 +

(fnk+1 fnk )

for x E

k=1

Now the partial sums SN (f ) is just

SN (f ) = fn1 +

(fnk+1 fnk ) = fnN

k=N

Define the function g(x) by,


f (x) = |fn1 | +

|fnk+1 fnk |

for x E

k=1

Now by Minikowskis inequality we have


kSN (g)kp kfn1 kp + k

N
1
X

|fnk+1 fnk |kp kfn1 kp +

k=1

N
1
X
k=1

1
2k

So the increasing sequences of partial sums kSn (g)kp is bounded above by kfn1 k + 1. Hence we have
Z
Z
Z
p
p
g <
|f | <
fp <
E

27

This implies that the series fnk converges almost everywhere. Now
N

X



|f fnN | = |S (f ) SN 1 (f )| =
fnk+1 fnk g


k=1

Hence by the Lebesgue dominated convergence thoerem we have


Z
lim kf fnk kpp =
lim (f (x) fnk )p = 0
k

E k

Hence fnk converges to f in Lp (E). Now fn is itself Cauchy, hence fn converges to f is in Lp (E).

f (x)g(x) dx is finite for any f (x) L2 . Prove

Exercise 4.9. Let g(x) be measurable and suppose


a

that g(x) L2 .
Z

Z
gdx < which implies that g L1 [a, b]. Let F =

Proof: If f = 1, then f L2 ([a, b]) so


a

then F is a bounded linear functional from L2 ([a, b]) to R. So there exists an M such that
(Z
)

gdx,
a

kF (f )k = sup
kf k2 =1

fg

< M,

f L2 ([0, 1])

Then by the Reisz Representation Theorem g must be in L2 ([0, 1])

.

Theorem (Riesz Representation) Let F be a bounded linear functional on Lp for 1 p < .


Then there exists a function g Lq such that
Z
F (f ) = f g.
We also have kF k = kgkq .
Proof: Just considering the finite dimensional case. Let be of finite measure. Then every bounded
measurable function is in Lp (). Define a set function on the measurable sets by (E) = F (E ). If
E is the union of a sequence En of disjoint measurable sets, define a sequence n = sgn F En and set
X
f=
n En
Then F is bounded and we have

X
|(En )| = F (f ) < ,
n=1

(En ) = F (f ) = (E)

n=1

Hence is a signed measure, and by construction it is absolutely continuous with respect to . By the
Radon-Nikodym Theorem, there is a measurable function g such that for each measurable set E we
have
Z
(E) =

g d
E

Since is always finite implies that g integrable. Now if is a simple function, the linearity of F and
of the integral imply that
Z
F () = g d
Since the left-hand side is bounded by kF kkkp we have g Lq . Now let G be the bounded linear
functional defined on Lp by
Z
G(f ) = f g d

28

Then G F is a bounded linear function which vanishes on the subspace of simple functions, which are
dense in Lp . Hence we must have G F = 0 in Lp . So for all f Lp , we have
Z
F (f ) = f g d
and by construction kF k = kGk = kgkq

.

Exercise 4.10. Let (X , M, ) be a measure space and let f be an extended real-valued M- measurable
function on X such that
Z
|f |p d < for p (0, ).
X
p

Show that lim {x : |f (x)| } = 0

Proof: First define the set E = {x X : f (x) }. Now notice that E E if > , also because
f Lp we have (E ) < 1 if is large enough, in particular (E ) = 0. Now
Z
Z
p {x : |f (x)| } = p
d
|f |p d
E

Hence we have
lim p {x : |f (x) }

|f |p d = 0

Since f Lp , then |f | L1 (X ), (E ) = 0 and the integral of an Lebesgue integrable function over


a set of measure zero is zero  .
5. Signed Measures
Remark: If (X , M) is a measure space, and if , are two measure defined on (X , M). and are
said to be mutually singular (), if there are disjoint stes A and B, in M such that X = A B
and (A) = (B) = 0. A measure is said to be absolutely continuous with respect to the measure ,
( << ), if (A) = 0 for each set A for which (A) = 0.

Exercise 5.1. Let be a measure and let , 1 , 2 be signed measure on the measurable space (X , A).
Prove:
(a) If and << , then = 0
(b) If 1 and 2 , then, if we set = c1 1 +c2 2 with c1 , c2 R such that is a signed measure,
thwn we have .
(c) If 1 << and 2 << , then, if we set = c1 1 + c2 2 with c1 , c2 R such that is a signed
measure, thwn we have << .
Proof: For part (a), if is a signed measure such that and << . There are disjoint measurable
sets A and B such that X = A B and ||(B) = ||(A) = 0. Then |(A)| = 0 so ||(X) = ||(A) +
||(B) = 0. Hence we have + = = 0 i.e. = 0.
For part (b), there are disjoint measurable sets Ai and Bi such that X = Ai Bi and (Bi ) = i (Ai ) = 0,
for i = 1, 2. Now X = (A1 A2 ) (B1 B2 ) and (A1 A2 ) (B1 B2 ) = . Now we have
(c1 1 + c2 2 )(A1 A2 ) = (B1 B2 ) = 0

(c1 1 + c2 2 )

For part (c), suppose 1 << and 2 << . If (E) = 0, then 1 (E) = 2 (E) = 0. Hence
(c1 1 + c2 2 )(E) = 0

(c1 1 + c2 2 ) <<

29

Exercise 5.2. Let be a positive measure and be a finite positive measure on a measurable space
(X , M). Show that if << , then for every  > 0 there is a > 0, such that for every E M with
(E) < , we have (E) < .
Proof: Suppose not, Then there is an  > 0 such that for every > 0, there is E M, such that
(E ) < , and (E ) . In particular, for every n 1, there is an En such that (En ) < n12 and
(En ) . Now we have

X
X
1
(En ) =
<
2
n
n=1
n=1
Let E = lim sup En , then (E) = 0. Now since << , we have (E) = 0. Now
(E) = (lim sup En ) lim sup (En ) 
But this implies that (En )  > 0, and hence (E) > 0, which is a contradiction. Therefore given
 > 0 there is a > 0, such that for every E M with (E) < , we have (E) <   .
Theorem (Hahn Decomposition) Let be a signed measure on the measurable space (X, M).
Then there is a positive set A and a negative set B such that X = A B and A B = .
Theorem (Jordan Decomposition) Let be a signed measure on the measurable space (X, M).
Then there are two mutually singular measure + and on (X, M) such that = + . Moreover,
there is only one such pair of mutually singular measures.

Exercise 5.3. Suppose (X , M) is a measurable space, and Y is the set of all signed measure on M
for which (E) < , whenevery E M. For 1 , 2 Y , define
d(1 , 2 ) = sup |1 (E) 2 (E)|
EM

Show that d is a metric on Y and that Y equipped with d is a complete metric space.
Proof: Since i are choosen such that i (E) < , then for any 1 , 2 Y and E M, we have
|1 (E) 2 (E)| < . So we have d : Y Y [0, ). Now to show d is a metric on Y we need to show
symmetry, positive definiteness and the triangle inequality. Clearly d(1 , 2 ) = d(2 , 1 ) by definition
of d. For the triangle inequality we have
d(, )

sup |(E) (E)|


EM

sup {|(E) (E)| + |(E) (E)|}




sup {|(E) (E)|} + sup |(F ) (F )|

EM

EM

F M

= d(, ) + d(, )
Now to show definiteness, if = , then |(E) (E)| = 0 for any E M, and so d(, ) = 0.
On the other hand if d(, ) = 0, then we have |(E) (E)| = 0. Let (A1 , B1 ), (A2 , B2 ) be Hahn
decompostions of , and respectively.
Case 1: If E A1 A2 , then (E) = + (E), and (Y ) = + (Y ), hence |(E)(E)| = |+ (E) + (E)|.
So we have + = + on A1 A2 .
Case 2, 3: If E A1 B2 , then we have (E) = (E) and (E) = + (E), hence
0 = |(E) (E)| = | (E) + (E)| = (E) + + (E)
Hence = + = 0 on E A1 B2 . If E A2 B1 , by the same proof we have the result + = = 0
on E A2 B1

30

Case 3: If E B1 B2 , then (E) = (E) and (E) = (E). So


0 = |(E) (E)| = | (E) + (E)|
and so = = 0 on E B1 B2 . So definiteness holds, therefore d is a metric on Y .
Now to show the metric space is complete. Let n be a Cauchy sequence. Then for any  > 0, there is
an N such that if m, n > N , we have
sup |vn (E) vm (E)| < 
EM

If particular, for a fixed set E, we have n is a Cauchy sequence in R. Hence there exists some (E) R,
such that n . By the uniform boundedness pricipal we know that is bounded, and hence
n in the metric d

Remark: The measure || is defined from the Jordan decomposition by, ||(E) = + E + E.
Theorem (Radon-Nikodym) let (X , M, ) be a -finite measure space, and let be a measure
defined on M which is absolutely continuous with respect to . Then there is a nonnegative measurable
function f such that for each set E on M we have
Z
(E) =
f d
E

The function f is unique in the sense that if g is any measurable function with this property then g = f
almost everywhere.
Proof: Only the finite case is considered. Let be finite then is a signed measure for each
rational number . Let (A , B ) be a Hahn decomposition for , and take A0 = X and B0 = .
Now B B = B A . So we have
( )(B B ) 0

( )(B B ) 0

hence we must have (B B ) = 0. Now there exists a measurable function f such that for each
rational we have f almost everywhere on A and f almost everywhere on B . Since B0 =
be an arbitrary set in M, and set
Ek = E (B(k+1)/N Bk/N )
Then E =

Ek , and this union is disjoint modulo null sets. Hence we have

k=1

(E) = (E ) +

(Ek ).

k=0

k+1
k
f
on Ek , and so
N
N
Z
k
k+1
f d
(Ek ).
(Ek )
N
N
Ek

Since Ek B(k+1)/N Ak/N , we have

Now since

k
k+1
(Ek ) (Ek )
(Ek ), we have
N
N
Z
1
1
(Ek ) (Ek )
f d (Ek ) + (Ek ).
N
N
Ek

) On E we have f = almost everywhere. If (E ) > 0, we must have (E ) > 0, since ( )(E


is positive for each . If (E ) = 0, we have (E ) = 0. Since << , for either case we have
Z
(E ) =
f d.
E

Hence we have
(E)

1
(E)
N

Z
f d (E) +
E

1
(E).
N

31

Z
Since (E) is finite and N arbitrary, we must have (E) =

f d.
E

The function f =

d
d

above is called the Radon-Nikodym derivative of with respect to .

Exercise 5.4. Suppose and are -finite measures on a measurable space (X , A), such that << ,
and << . Prove that


d

xX :
=1
= 0.
d
Proof: First notice that if E X , such that ( )E = 0, then we have (E) = (E). But we have
<< , hence if ( )E = 0, then (E) = 0 = (E). Conversely if (E) = (E) and << ,
then (E)
n (E) = 0, and
o so = 0 thus (E) = 0. So if (E) = (E), then (E) = (E) = 0. Now
let E = x X :

but

d
d

d
d

= 1 and consider (E). By the Radon-Nikodym theorem we have


Z
Z
d
(E) =
d =
d
d
E
E

= 1 on E, and so
Z

Hence (E) =

n

xX :

d
d

(E) =
o
=1
=0

d
d =
d

Z
d = (E)
E

.
Exercise 5.5. Let and be two measure on the same measurable space, such that is -finite and
is absolutely continuous with respect to .
(a) If f is a nonngeative measurable function, show that
Z
Z  
d
f d = f
d
d
(b) If f is a measurable function, prove that f is integrable with respect to , if and only if f

d
d

is

integralble with respect to , and in this case, part (a) still holds.
Proof: For part (a), let E be a measurable set and let f = E . Suppose that h =
Z
Z
Z
Z
Z
f d = E d = (E) =
h d = hE d = f h d.

d
d

exists. Then

So the equality holds for charactersitc functions. Let f = be a simple function, then by the above we
have
Z
Z
d = h d.
Now let f be a nonnegative measurable function. There there exists a monotone sequence of simple
functions n such that 0 n f and n f almost everywhere. Applying the Monotone Covergence
theorem, we have
Z
Z
Z
Z
f d = lim

n d = lim
n h d = f h d  .
n
R +
R
For part (b), f is -integrable if and only if f d f d is finite. Now by part (a) we have
Z
Z
Z
Z
f + d = f + h d and
f d = f h d.
n

So we have f is -integrable if and only if f is -integrable

.

Theorem (Lebesgue Decomposition) Let (X , M, ) be a -finite measure space and a -finite

32

measure defined on M. Then we can find a measure 0 , singular with respect to and a measure 1
absolutely continuous with respect to , such that = 0 + 1 . Furthermore, the measures 0 and 1
are unique.
6. Topological and Product Measure Spaces
Exercise 6.1. Let L be a normed space. Then every weakly bounded set X is bounded.
Proof: Let : L L , by (x)(f ) = f (x), where x L, f L . Now X is a Banach space and
(X) is a family of bounded linear functionals on X , and for each f L we have
sup{(x)(f ) : x X} = sup{f (x) : x X} <
Then from the uniform boundness principle we have
sup{kxk : x X} = sup{k(x)k : x X} <
Therefore every weakly bounded nonempty set of a normed space is bounded

.

Exercise 6.2. Suppose that A is a subset in R2 . Define for each x R2 , p(x) = inf{|y x| : y A}.
Show that Br = {x : p(x) r} is a closed set for each nonnegative r. Is the measure of B0 equal to the
outer measure of A?
Proof: Let z (Br ), and let  > 0. Then there is x Br such that |x z| < . So we have
p(z)

inf{|z y| : y A}

inf{|z x| + |x y| : y A}
 + inf{|x y| : y A}
 + r.
This is for all  > 0, therefore p(z) r which implies z Br thus Br is closed. Now B0 = A A.
First by definition of p(x) we have for any x A, p(x) = 0. Hence x B0 , Now suppose that x A,
then for any  > 0, there is a y A such that |x y| < . Therefore we have
p(z) = inf{|z y| : y A} = 0

x B0 ,

and so A B0 . Now suppose x B0 . Then inf{|x y| : y A} = 0, so for every  > 0 there is a y A


such that |x y| < . So x A, therefore we have B0 = A = A A. Now
(A) (B0 ) = (A A) = (A ) + (A) = (A) + (A)
Since A is open and A is measurable. Therefore (A) = (B0 ), if and only if (A) = 0

.

Exercise 6.3. Prove that an algebraic basis in any infinite-dimensional Banach space must be uncountable.
Proof: Let V be an infinite-dimensional Banach space over F, and suppose {xn }nN is a countable
Hamel basis. Then v V if any only if there exists ai F such that
v=

k
X

ai xi

i=1

for some xi {xn }. Now let hxi i denote the span of xi , then we have
[
V =
h{xn }kn=1 i
kN

But this implies that V is a countable union of proper subspace of finite dimension. Which implies
that V would be of first category, since every finite dimensional proper subspace of a normed space is
nowhere dense. Which is a contradiction to the Baire Category Theorem. Therefore any basis for an

33

infinite-dimensional Banach space must be uncountable

Theorem (Hahn-Banach) Let p be a real-valued function defined on the vector space X satisfying
p(x + y) p(x) + p(y) and p(x) = p(x) for each 0. Suppose that f is a linear functional defined
on a subspace S and that f (s) p(s) for all s S. Then there is a linear function F defined on X
such that F (x) p(x) for all x, and F (s) = f (s) for all s S.

Exercise 6.4. Let be a finite Borel measure on the real line, and set F (x) = {(, x]}. Prove
that is absolutely continuous with respect to the Lebesgue measure if and only if F is an absolutely
continuous function. In this case show that its Radon-Nikodym derivative is the derivative of F , almost
everywhere.
Proof: () First suppose that << . Let (E), then there exists an open set O, such that E O
and (O) < . Now O being open, there are disjoint intervals (xk , yk ), such that
[
X
O=
(xk , yk ),
(O) =
(yk xk ) < 
k=1

k=1

Since << , there exists a delta such that if (O) < , then (O) < . So we have
X
X
|F (yk ) F (xk )| =
(xk , yk ) <
k=1

k=1

So F (x) is an absolutely continuous function.


() Suppose that F (x) is absolutely continuous. Then we have
X
X
 > 0 > 0 s.t.
|yk xk | <

|F (yk ) F (xk )| < 


k=1

k=1

Choose such disjoint intervals (xk , yk ) and call the union of these intervals O, then we have (O) < .
Now by definition of F (x), we have
X
(O) =
|F (yk ) F (xk )| <
k=1

and so << .
To see that F is Radon-Nikodym derivative, we know that since F is absolutely continuous we have
that F 0 (t) exists almost everywhere so
Z x
(, x] = F (x) =
F 0 (t) d(t)

We also have that


x

Z
(, x] =

d =

d
d

which implies that


Z

(, x] = F (x) =

F 0 (t) d(t) =

d
d


d


d
Hence by the Radon-Nikodym theorem we know that F =
almost everywhere.
d
0

Theorem (Tonnelis) Suppose (X Y, (A B), ) is the product space of two -finite measure
spaces, and f : X Y [0, ] and f (x, y) be a nonnegative measurable function in the product
measure, then
Z
F1 (x) =
f (x, ) d is A measurable of x X
Y

34

Z
f (, y) d is B measurable of x Y

F2 (y) =
X

and
Z

Z
f d() =

X Y

Z
F1 d =

F2 d
Y

i.e., the iterated integrals is equal to the the integral in the product space


Z Z
Z Z
Z
f (x, y) d d =
f (, y) d d =
f (x, y) d( ).
X

X Y

Theorem (Fubinis) Suppose(X Y, (A B), ) is the product space of two -finite measure
spaces, and f : X Y [0, ] and f (x, y) be an integrable function in the product space, then


Z Z
Z Z
Z
f (x, y) d d =
f (, y) d d =
f (x, y) d( ).
X

X Y

Theorem (Fubini-Tonelli) Suppose (X Y, (A B), ) is the product space of two -finite


measure spaces. Let f be an extended real-valued (A B) measurable function on X Y. If either


Z Z
Z Z
|f | d d < or
|f | d d <
X

then f is -integrable, furthermore the iterated integrals are equal to the product integral.

Exercise 6.5. Let f be a real valued measurable function on the finite measure space (X , M, ). Prove
that the function F (x, y) = f (x) 5f (y) + 4 is measurable in the product measure space (X X , (M
M), ), and that F is integrable if and only if f is integrable.
Proof: First since f (x) is measurable, we have both sections F (x0 , y), and F (x, y0 ) as measurable
for each fixed x0 , y0 . Now for x X we have F (x, x) = 4(f (x) 1), which is measurable. Having
F (x, y) being measurable on each section, and the diagonal is enough for F (x, y) to be measurable in
the product space.
Z
Now let f be integrable, hence |f | is integrable, so let M =
|f (x)| dx, now we have
X

Z Z

Z
|f (x) 5f (y) + 4| dxdy

M + 5|f (y)|(X ) + 4(X ) dy


X

M (X ) + 5M (X ) + 4(X )2

4(X )(M + (X )) <


Z Z

|f (x) 5f (y) + 4| dydx <


=

by the same computation

Then by Fubini-Tonelli theorem F (x, y) is integrable. Now suppose that F (x, y) is integrable, then by
Fubinis theorem we have that the iterations are equal, but this is true if and only if f (x) is integrable
.
Theorem (Stone-Weierstrass) Let X be a compacct space and A an algebra of continuous realvalued functions on X that separates the points of X and contains the constant functions. Then given
any continuous real-valued function f on X and any  > 0 there is a function g A such that for all
x X we have |g(x) f (x)| < . In other words, A is a dense subset of C(X).
Theorem (Closed Graph) Let A be a linear transformation on a Banach space X to a Banach space
Y . Suppose that A has the property that, whenever xn is a sequence in X that converges to some point
x and Axn converges in Y to a point y, then y = Ax. Then A is continuous.

35

Exercise 6.6. Let (X , A, ) and (Y, B, ) be the measure spaces given by


X = Y = [0, 1]
A = B = ([0, 1])
be the Lebesgue measure on R, and the counting measure.
Consider the product measure space (X Y, (A B)), and its subset E = {(x, y) X Y : x = y}
(1) Show that E
R R (A B)
R R
(2) Show that X Y E d d 6= Y X E d d.
(3) Explain why Tonellis theorem is not applicable.
Proof: For (1) First notice that the following sets
 


k1 k
k1 k
,

,
Ak =
n
n
n
n
are measurable. Now let define En as follows
n
[
En =
Ak ,
k=1

Then the sets En are measurable as they are countable union of measurable sets. Then the set E is
given by

\
En = {(x, y) X Y : x = y}
E=
n=1

is measurable since is a countable intersection of measurable sets.


For (2) by a direct computation we have
Z Z
Z
E d d =
X

Z
E d d =

(E)d =

d = 1
0

Z
(E) d =

and
Z Z

0 d = 0
0

Tonellis theorem is not applicable because the measure space (Y, B, ) is not -finite

.

You might also like